sin pi/3 __ __ pi/6 = 1/2(sin pi/2 + sin pi/6)

I think I’m just supposed to fill in the blank? (question off of a p e x) please give explanation!

Answers

Answer 1

Notice that

π/2 = π/3 + π/6

π/6 = π/3 - π/6

Recall the angle sum identities for sine:

sin(x + y) = sin(x) cos(y) + cos(x) sin(y)

sin(x - y) = sin(x) cos(y) - cos(x) sin(y)

By adding these together, we get

sin(x + y) + sin(x - y) = 2 sin(x) cos(y)

==>   sin(x) cos(y) = 1/2 (sin(x + y) + sin(x - y))

Now take x = π/3 and y = π/6 :

sin(π/3) cos(π/6) = 1/2 (sin(π/2) + sin(π/6))

So the blank should be filled with cos.


Related Questions

Sweatshirts at the school store cost $30. They currently sell about 4 shirts per month. They have decided to decrease the price of the shirts. They found the for each each $1.50 decrease, they will sell 2 more shirts per month. How many decreases in prices will give the max value?

A. 30

B. 9

C. 363

D. 16.50

Answers

a club usually sells 1200 shirts a year at $20 each. A survey indicates that
for every $2 increase in price, there will be a drop of 60 sales a year.
What price they should charge for each shirt to maximize the revenue?
:
Let x = no. of $2 increases
also
Let x = no. 60 shirt sales reductions
:
Price = (20 + 2x)
No. of shirts = (1200 - 60x)
:
Revenue = price * no. of shirts sold, therefore:
R = (20 + 2x)*(1200 - 60x)
FOIL
R = 24000 - 1200x + 2400x - 120x^2
Arranges as a quadratic equation
R = -120x^2 + 1200x + 24000
Find the axis of symmetry to find the price for max revenue: x = -b/(2a)
In this equation; a=-120; b=1200
x = %28-1200%29%2F%282%2A-120%29
x = %28-1200%29%2F%28-240%29
x = +5 ea $2 increases
and
5*60 = 300 reduction in shirt sales:
:
Price for max sales; 20 +2(5) = $30, will sell 1200 - 300 = 900 shirts
:
Max revenue: 30 * 900 = $27,000

If a school camp has enough food for 150 students over 5 days
How long would the food last for is only 100 students are there

Answers

Answer:

It is over 7 days

hope it helps

[tex](32)^{1/5}[/tex]

Answers

Answer:

2

Step-by-step explanation:

[tex] {32}^{ \frac{1}{5} } [/tex]

[tex] = \sqrt[5]{32} [/tex]

[tex] = \sqrt[5]{2 \times 2 \times 2 \times 2 \times 2} [/tex]

= 2 (Ans)

A cylinder has a volume of 132x cubic feet and height of 4 feet. What is the area of the base?
* Square feet

Answers

Answer:

33π square feet

Step-by-step explanation:

Volume = base area × height

132π = base area × 4

or, base area = 132π/4

or, base area = 33π

Answered by GAUTHMATH

is 1/5 a rational or irrational number (20 points pls help)

Answers

Answer:

rational

Step-by-step explanation:

Answer:

it's Rational Number

Step-by-step explanation:

since each integer can't be written in the fraction form or for this example it would be thus 5 so it would be a rational number

50 points and brainly
Trinas you company charges $0.86 per pound to ship toys. Part A: Write an equation to determine the totals cost, c, to ship p pounds of toys. Use your equation to determine the cost of shipping 2 pounds of toys. Path B: if the company reduces the cost to ship toys by $0.05 per pound, write an equation to determine the total cost, c, to ship p pounds of toys with the reduced cost.

Answers

Answer:

Part A:

c = .86*p

c =1.72 for 2 lbs

Part B:

c =.81p

Step-by-step explanation:

Part A:

Total cost = cost per pound * number of pounds

c = .86*p

Let p = 2

c = .86*2

c =1.72

Part B:

Total cost = cost per pound * number of pounds

c = .(.86-.05)*p

c =.81p

(3x)2 – 7y2, when x = 3 and y = 2

Answers

[tex]3 \times 3 \times 2 - 7 \times 2 \times 2 \\ 18 - 28 \\ - 10[/tex]

Answer:

115

Step-by-step explanation:

(3x)^2 – 7y^2

Plug in:

(3×3)^2 – 7×2^2

9^2 - 14^2

81 - 196

115

Hope this helped.

what number is smallest 2,-3,-10,-5,-2-1

Answers

Answer:

2,-3,-10,-5,-2-1

The smallest number is -10

Answer:

it's -10

Step-by-step explanation:

Well, if you look at the number line, numbers start from the left going to the right and the ones before the 0 ( the negative ones) are the smaller ones. And here if you arrange them from the smallest, -10 is the smallest and 2 is the biggest.

hope this helps

This equation shows how the total number of necklaces Denise owns is related to the amount of money she spends on additional necklaces.

n = d + 74

The variable d represents the amount of money she spends on additional necklaces, and the variable n represents the total number of necklaces she owns. With $16 to spend on new necklaces, how many total necklaces can Denise own?

Answers

Answer:

16+74

Step-by-step explanation:

substitute 16 into the equation

Answer: n=90

d=16 so n= 16+74

please mark brainliest

3. In a school, there are 450 students in total. If 2⁄3 of the total students are boys, find the number of girls in the school. What percent of the students are boys? Show your work or explain how you calculated your answer

Answers

Answer:

150 girls

66.7%

Step-by-step explanation:

2/3 × 450 = 300 boys

450 - 300 =150 girls

second question:

300/450 × 100

=66.66666667

=66.7% Students are boys

There are 150 girls in the school, and approximately 66.67% of the students are boys.

What percent of the students are boys

let's break this down step by step:

1. Total number of students: 450

2. Boys make up 2/3 of the total students.

Number of boys = (2/3) * 450

= 300 boys

3. To find the number of girls, subtract the number of boys from the total:

Number of girls = Total students - Number of boys

= 450 - 300

= 150 girls

Now, let's calculate the percentage of students that are boys:

Percentage of boys = (Number of boys / Total students) * 100

= (300 / 450) * 100

= 66.67%

So, there are 150 girls in the school, and approximately 66.67% of the students are boys.

Learn more about percentage at https://brainly.com/question/24304697

#SPJ3

Marking Brainliest. Can someone please how to do this with a fairly simple explanation? I'm not sure what to do.

Answers

Answer: See the diagram below

Explanation:

In the LS column, you'll have these steps

3x - 1

3*4 - 1

12 - 1

11

Effectively, we replaced x with 4 and then simplified using PEMDAS.

And in the RS column, you'll have these steps

x + 7

4 + 7

11

We get the same thing at the bottom of each column. This shows that we end up with 11 = 11 after simplifying both sides. Therefore, we've confirmed that x = 4 is the solution to 3x - 1 = x + 7

PLEASE HELP!!!!!!!
Which terms are like terms in the following expression? 3x + 3xy – 2x + 5y + 5x2
A. 3x and -2x
B. 3x, -2x and 5x2
C. 3x, 3xy, -2x and 5y
D. All terms are alike.

Answers

Answer:

C

Step-by-step explanation:

A the answer is A because they have the same ending

Find the missing value.
Hint: Use the number line to find the missing value.
了。
-(-2)
{

-10
-5

15
0
5
10
-15

Answers

Answer:

-5

Step-by-step explanation:

-5 -(-2)

=-7

I hope this helped!


I NEED HELP ASAP ITS FOR MY SCHOOL WORK AND I CANT FIGURE THIS OUT

Answers

Answer:

28.

      megabyte- [tex]10^6[/tex]

      terabyte- [tex]10^{12}[/tex]

      gigabyte- [tex]10^9[/tex]

       For [tex]10^6, 10^9, 10^{12},[/tex] and [tex]10^{15}[/tex], put the 1, and add the exponent # of 0s.

29.

       The cube is 6×6×6   or    216 in. It would be 6³, because the 6 is multiplied 3 times.

30.

 a. 10

 b. 5120

I hope this helps!

pls ❤ and mark brainliest pls!

Answer:

Question 28

megabyte: 10⁶

terabyte: 10¹²

gigabyte: 10⁹

Question 29

Question 30

a. 20

b. 5,120

Help me please. I don't understand geometry.

Answers

Answer: ~p ~q.

Explanation: The inverse is "If ~p then ~q." Symbolically, the inverse of p q is ~p ~q.

a horizontally laid standard weight pipe has a diameter of 10 inches and is 12 feet long. The pipe is filled 9 inches high with water. How many gallons of water does the pipe hold? (Round your answer to the nearest gallon.) Please help.

Answers

Answer:

d isósceles hijo muy graciosa ya no me acuerdo de nada y tu q ases hablas con el

Find C to two decimal places and find the measurement of angle be use your calculator

Answers

Let's solve

P=8B=5

Using Pythagorean theorem

[tex]\\ \sf\longmapsto H^2=P^2+B^2[/tex]

[tex]\\ \sf\longmapsto c^2=8^2+5^2[/tex]

[tex]\\ \sf\longmapsto c^2=64+25[/tex]

[tex]\\ \sf\longmapsto c^2=89[/tex]

[tex]\\ \sf\longmapsto c=\sqrt{89}[/tex]

[tex]\\ \sf\longmapsto c\approx 9.22[/tex]

Please show your work
[tex]1\frac{2}{3} + 2\frac{3}{4}[/tex]

Answers

Answer:

[tex]→1 \frac{2}{3} + 2 \frac{3}{4} \\ = \frac{5}{3} + \frac{11}{4} \\ = \frac{20 + 33}{12} \\ = \frac{53}{12}\\ =\boxed{4\frac{5}{12}}✓[/tex]

53/12 is the right answer.

Steps:

1 2/3 + 2 3/4 can also be written as...

5/3 + 11/4

The LCD is 12 so we make both fractions have a denominator of 12.

That gives us 20/12+33/12

Now you can add the numerators (but leave the denominator the same)

That gives us 53/12, but our simplified version would give us 4 5/12 as our final answer.

Answer:

[tex]4\frac{5}{12\\}[/tex]

Please mark brainliest if that helps!

simplify (4+√3)(4-√3)

Answers

Answer:

13

Step-by-step explanation:

(4+√3)(4-√3) = (4)^2-(√3)^2 =16-3 = 13

Answer:

Step-by-step explanation:

(a + b)(a -b) =a² - b²

a = 4 and  b = √3

(4 + √3)(4 - √3)= 4² - (√3)²

                        = 16 - √3*√3

                        = 16 - 3

                        =  13

Which of the following are valid probability distributions?

Answers

Answer:

Step-by-step explanation:

calculate the speed, in ms-¹ of vehicle A and of vehicle B​

Answers

9514 1404 393

Answer:

A: 0.32 m/sB: 0.56 m/s

Step-by-step explanation:

The speed is the ratio of the change in distance to the corresponding change in time.

Vehicle A moves from a position of 12 m to one of 28 m in 50 seconds, so its speed is ...

  A = (28 -12)/50 m/s = 16/50 m/s = 0.32 m/s

Vehicle B moves from 0 to 28 m in 50 seconds, so its speed is ...

  B = (28 m)/(50 s) = 0.56 m/s

If the campers at Camp Grilled Meat prefer hot dogs to hamburgers by a ratio of 4:3, and they need 500 hot dogs, how many hamburgers will they need?

Answers

Step-by-step explanation:

jsndmdmsmsnananansndbdbnd

Can someone help me on this math question its confusing ill give brainly when right!!!

Answers

Answer:

x=5

Step-by-step explanation:

x=[tex]\sqrt{25}[/tex]

x=5

Given the quadratic equation (x + p)²=49,where p is a constant,find the values of p if one of the roots of the equation is 4.
Pls help me,will mark as the brainliest.​

Answers

Answer:

-11, 3

Step-by-step explanation:

We know one of the roots are

[tex](x+ 4) {}^{2} = 49[/tex]

[tex]x + 4 = - 7[/tex]

[tex]x + 4 = 7[/tex]

[tex]x = - 11[/tex]

or

[tex]x = 3[/tex]

(10x²+19x+6) ÷(5x+2)​

Answers

Answer:

2x + 3

Step-by-step explanation:

(10x² + 19x + 6) ÷ (5x + 2)​ =

= (10x² + 15x + 4x + 6) ÷ (5x + 2)​

= [5x(2x + 3) + 2(2x + 3)] ÷ (5x + 2)​

= [(5x + 2)(2x + 3)] ÷ (5x + 2)​

= 2x + 3

Answer:

2x+3 is your answer

X
45°
454
Find the value of x.
A.
B.
3.2
2
C. 3√2
D. 33
Save and Fyit

Answers

The answer to this question is C

The library has 2,730 books. If each shelf can hold 30 books, how many shelves will the library need to hold all of its books?

Answers

91 shelves are needed in the library

Answer:

2730÷30=91

Step-by-step explanation:

take the number of books and divide it by the number of books each shelve can hold and ull get ur answer

15. Mary was given data comparing students’ mark in math class and the number of classes missed. She plotted the data on the graph below and drew a line of best fit. Do you agree with Mary’s drawing of the line of best fit? Justify your answer

Answers

nobody is helping me out rip

Which of the following is equivalent to 0.00451?
A. 4.51x10^-3
B. 45.1x10^-3
C. 0.451x10^-3
D. 4.51x10^-2

Answers

Step-by-step explanation:

0.00451

[tex] = \frac{451}{100000} [/tex]

[tex] = \frac{4.51}{1000} [/tex]

[tex] = \frac{4.51}{ {10}^{3} } [/tex]

[tex] = 4.51 \times {10}^{ - 3} (option \: a)(ans)[/tex]

The required scientific notation  is A. 4.51x10⁻³  is equivalent to 0.00451. It's in scientific notation where 4.51 is between 1 and 10, and -3 is the exponent representing the decimal shift.

In scientific notation, a number is written in the form of "a × 10ⁿ" where "a" is a number between 1 and 10 (including 1 but excluding 10), and "n" is an integer exponent.

The goal is to represent the number in a more compact form using powers of 10.

0.00451

To express this in scientific notation, we need to find "a" and "n" such that "a × 10ⁿ" equals 0.00451.

The option A, 4.51 × 10⁻³ is the correct choice because:

"a" is 4.51, which is between 1 and 10.

"n" is -3, as we are shifting the decimal point three places to the left to convert 0.00451 to 4.51 × 10⁻³.

Therefore, 0.00451 is equivalent to  scientific notation option A. 4.51 × 10⁻³, which is option A.

learn more about  scientific notation here

brainly.com/question/19625319

#SPJ3

Guided Practice


Find the first, fourth, and eighth terms in the sequence.

an=−5 · 3n−1a subscript n baseline equals negative 5 times 3 superscript n minus 1 baseline


A.
–15; –405; –32,805


B.
5; 135; 10,935


C.
–5; –135; –10,935

Answers

Answer:

C.

–5; –135; –10,935

Step-by-step explanation:

Answer:

C. -5; -135, -10,935

Step-by-step explanation:

...............................................................................................................................................

Other Questions
Steve, a high school student, is attending a biology class. When the teacher asks each student in the class to recall the function of the sympathetic nervous system, all the students who were asked the question before him incorrectly recall the function. Despite knowing the correct answer, Steve repeats the answer given by other students because of group pressure. This scenario exemplifies the concept of __________. Group of answer choices Find the attached question. Who was the first president? The function (x) = 6x is vertically shrunk by a factor of and translated 9 units in the negative y- direction. Select the correct graph of the resulting function. movement of water molecules without the use of energy What is the result of (305.120 + 267.443) x 0.50? How many answers can be written based on the principle of significant digits? Suppose you borrowed $10,000 via a pure discount loan at 6.25 percentinterest per year and were going to pay back both principal and interest infive years. How much interest would you pay? Directions: complete each statement by filling in the blank with the correct past participle form of the given verb. 1. Mario y Javier (parar) en el estadio.2. Ella esta (preparar) para el examen 3. Toda la comida esta (quemar) 4. El banco esta (Cerrar) What led President John F. Kennedy to blame Soviet premier Nikita Khrushchev for causing a threat to world peace?the Soviet role in the overthrow of Fulgencio Batistathe building of Soviet nuclear missile sites in Cubathe Soviet enforcement of a naval blockade of Cubathe Soviet role in the failure of the Bay of Pigs invasion The initial development of textile mills was made possible, in part, by the abilityto move coal and raw materials bya) railroad b) truckc) horse and wagond) canals For a certain company, the cost for producing x items is 40x+300 and the revenue for selling x items is 80x0.5x2. The profit that the company makes is how much it takes in (revenue) minus how much it spends (cost). In economic models, one typically assumes that a company wants to maximize its profit, or at least wants to make a profit!Part a: Set up an expression for the profit from producing and selling x items. We assume that the company sells all of the items that it produces. (Hint: it is a quadratic polynomial.)Part b: Find two values of x that will create a profit of $300.The field below accepts a list of numbers or formulas separated by semicolons (e.g. 2;4;6 or x+1;x1). The order of the list does not matter. To enter a, type sqrt(a).Part c: Is it possible for the company to make a profit of $15,000? Your new motorcycle weighs 2450 N.What isits mass in kilograms? i understand the answer is 250kg because mass times gravity thing but why isnt the answer negative cause gravity pulls down so wouldnt be Negative answer not a positive answer i was taught FW is always negative why isnt it negative here Work out the length x. 14 cm 7 cm A study by researchers at a university addressed the question of whether the mean body temperature of an animal is 98 6F Among other data, the researchers obtained the body temperatures of 109 healthy animals. Suppose you want to use those data to decide whether the mean body temperature of healthy animals is less than 98.6F.Required:a. Determine the null hypothesisb. Determine the alternative hypothesis Dodi bicycles 14mph with no wind. Against the wind, Dodi bikes 10mi in the same time it takes to bike 20mi with the wind. What is the speed of the wind? Select all that apply Which of the following are characteristics of frequency tables? Multiple select question. They can be used for qualitative data. They can be used for quantitative data. No observation can fit into more than one class. The percentage of observations in each class is provided. 45 girls and 30 boys ratio in the lowest terms pls help 10 points each 2 question due in 30 mins Laura is taking her family to an ice show. Each ticket costs $15, and programscost $6 each. She has $97. She buys 5 tickets and wants to figure out thegreatest number of programs she can buy.If x = number of tickets and y= number of programs, which of the followinginequalities could you use to find the answer to the problem?A. X+ y = 97B. 15 + 6y s 97C. 15x + 6ys 97D. 15x + 6y2 97 How many electrons are shown in the following electronconfiguration: 1s22s22p63s 23p64s23d104p65s24d105p66s2 ?Express your answer numerically as an integer.